stackoutawinner
Thanks Received: 2
Jackie Chiles
Jackie Chiles
 
Posts: 45
Joined: June 30th, 2009
 
This post thanked 1 time.
 
trophy
First Responder
 

Diagram

by stackoutawinner Sat Aug 01, 2009 9:28 pm

This game came up in a recent study hall I attended. As part of my own prep, I not only want to understand a game, but to be able to explain it completely. For this reason, I'm posting a solution to this game along with a bonus question. The solution is also attached.
PT30, Game 2, Stacks Solutions.pdf
(69.07 KiB) Downloaded 152845 times
(Moderator note: PT30, Game 2, Stacks Solutions.pdf)

Mind you the bonus Q is not a real LSAT question, it's one that I made up.

The Bonus Q:
If Pasquale leaves three messages, then all of the following could be true EXCEPT:
A. Liam leaves the 4th message
B. Pasquale leaves the 5th message
C. Hildy leaves the third message
D. Theodore leaves the fourth message
E. Hildy leaves the sixth message
 
dan
Thanks Received: 155
Atticus Finch
Atticus Finch
 
Posts: 202
Joined: March 10th, 2009
 
 
 

re: PT30, S1, G2 SOLUTIONS + bonus Q The six messages of an

by dan Mon Aug 03, 2009 9:33 am

Stacks, thanks for this fantastic solution. This is exactly the sort of work that will lead to improved games performance. Mastering any one game (as you've done here) is more valuable than kind of being familiar with 10 games. Really great work.

A small point... in the answer to your bonus question, you mention "since there is no G, H can't be in 1." Maybe I'm missing something? This would be true if there were no P, but no G? Let me know if I've overlooked something.

Thanks again Stacks!

dan
 
stackoutawinner
Thanks Received: 2
Jackie Chiles
Jackie Chiles
 
Posts: 45
Joined: June 30th, 2009
 
 
trophy
First Responder
 

Re: PT30, S1, G2 SOLUTIONS + bonus Q The six messages of an

by stackoutawinner Mon Aug 03, 2009 3:32 pm

Great question Dan!

I don't think you missed anything, I think I got turned around in typing. Suppose we have 3 P's, and we put H in the first slot. Because F is not appearing, the rule that says all of P must come before T does not come into play. I think somewhere when I was writing I confused the H first means P last rule into H first into G last... which is a mental mistake that can cost points. This would explain why I said if no G, then no H in the first slot. The actual rule is that if there's no P, then there's no H in the first slot, and that rule doesn't even come into play.

My bad. Good catch!
 
derb284
Thanks Received: 0
Forum Guests
 
Posts: 1
Joined: June 04th, 2010
 
 
 

Re: PT30, S1, G2 SOLUTIONS + bonus Q The six messages of an

by derb284 Fri Jun 04, 2010 10:19 pm

The second question on this game (#7) was driving me nuts, apparently I was reading it wrong (as): The first and last messages on the answering machine could be the first and second messages left on the machine (I didn't get that the question meant second as in the second message left by the same person, not as in the second slot). In other words I interpreted the question as, the first, second, and sixth messages could all have been left by who? In which the correct answer could have been Pasquale, P P H L T P. Make sure you read the question very carefully so you don't make the same mistake I did and end up scratching your head for a half hour trying to figure out why your right answer is in fact wrong! Most of my mishaps occur because I cannot fully understand what the question is actually asking...
 
savannajolie
Thanks Received: 0
Forum Guests
 
Posts: 5
Joined: March 26th, 2010
 
 
 

Re: PT30, S1, G2 SOLUTIONS + bonus Q The six messages of an

by savannajolie Tue Oct 26, 2010 1:58 pm

Q6. Dan said: F followed by P then T and then by P again, but in my book its followed by H then P then L. Do you think it was a typo in my book? Please advise. Thanks
 
savannajolie
Thanks Received: 0
Forum Guests
 
Posts: 5
Joined: March 26th, 2010
 
 
 

Re: PT30, S1, G2 SOLUTIONS + bonus Q The six messages of an

by savannajolie Tue Oct 26, 2010 2:13 pm

Correction: Stackawinner wrote it not Dan.
User avatar
 
ManhattanPrepLSAT1
Thanks Received: 1909
Atticus Finch
Atticus Finch
 
Posts: 2851
Joined: October 07th, 2009
 
 
 

Re: PT30, S1, G2 SOLUTIONS + bonus Q The six messages of an

by ManhattanPrepLSAT1 Thu Oct 28, 2010 2:16 pm

Hi savannajolie!

If you're looking at answer choice (A) from Q6, then you're book does not have a typo. My book has the same order that you listed - F, P, T, H, P, L. I'm not sure that the discussion above is really about that answer choice anyway. Your book seems to be fine.

Good luck!
 
savannajolie
Thanks Received: 0
Forum Guests
 
Posts: 5
Joined: March 26th, 2010
 
 
 

Re: PT30, S1, G2 SOLUTIONS + bonus Q The six messages of an

by savannajolie Fri Oct 29, 2010 8:40 am

Thanks MS,

I was referencing the downloaded solutions and explanations for Q 6 that a poster provided. The explanation stated a diff order for this question.Thus, stumping me.Please provide a detailed explanation as to why- A is not the credited answer. Thanking you in advance.
User avatar
 
ManhattanPrepLSAT1
Thanks Received: 1909
Atticus Finch
Atticus Finch
 
Posts: 2851
Joined: October 07th, 2009
 
 
 

Re: PT30, S1, G2 SOLUTIONS + bonus Q The six messages of an

by ManhattanPrepLSAT1 Fri Oct 29, 2010 8:56 pm

I don't think stackoutawinner meant to imply that P immediately followed T, but that there was a second P that followed T at some point.

So in your answer choice (A) it goes F, P, T, H, P, L. In which case it goes F, P, T and then after that at some point comes another P, which would be grounds for eliminating answer choice (A). This would violate the 5th constraint.

Does that make sense?
 
giladedelman
Thanks Received: 833
LSAT Geek
 
Posts: 619
Joined: April 04th, 2010
 
 
 

Re: Diagram

by giladedelman Sat Apr 23, 2011 6:03 pm

Whoops, looks like the attachment got lost somehow! Here's the diagram I drew. Let me know if you have any more questions.
Attachments
PT30, S, G2 - Stacks Solutions - ManhattanLSAT.pdf
(99.66 KiB) Downloaded 418 times
 
theonlyrij
Thanks Received: 1
Vinny Gambini
Vinny Gambini
 
Posts: 13
Joined: March 26th, 2011
 
 
 

Re: Diagram

by theonlyrij Thu Sep 08, 2011 9:55 pm

Hi,

Could someone please explain this game to me? I got the set up correct, but I do not understand the inferences from the diagram. Why does there need to be a minimum of 4 people and why does it have to P,T,H,L?

Thanks!
 
timmydoeslsat
Thanks Received: 887
Atticus Finch
Atticus Finch
 
Posts: 1136
Joined: June 20th, 2011
 
 
trophy
Most Thanked
trophy
First Responder
 

Re: Diagram

by timmydoeslsat Thu Sep 08, 2011 11:33 pm

theonlyrij Wrote:Hi,

Could someone please explain this game to me? I got the set up correct, but I do not understand the inferences from the diagram. Why does there need to be a minimum of 4 people and why does it have to P,T,H,L?

Thanks!


Definitely the key to the game in my opinion.

The rules are:

H1 ---> P6

G ---> F and P

F ---> All P > T

P ---> All H> L

We also know that at most only one person can leave more than a message.

We also know that the maximum amount that person could leave would be 3.

Notice what happens if we do not have L. We know via the contrapositive that we do not have P. We then know that we do not have F and G. We only have H and T left. We know that could not work because how could we get 6 messages from two people? The numbers do not work.

Same with H. Without H, we know there is no P, then we know there is no F and no G. Only T and L left to place six messages. Will not work.

Same with P. Without P, we do not have F and we do not have G.

That leaves us with 3 variables: H, L, and T. Even with three variables, the numbers do not work.

We could have one of those variables go 3 times, the maximum amount. So we now have exhausted the only variable that can go more than once and we have three spaces left to place with only 2 variables that can go only once. Will not work.

If T was not used, then F could not be used, and then G could not be used. Three variables, six messages. Will not work.

So this means that we have to have at least 4 variables to make this game work. And we also know that we have to have P, H, L, and T.
 
eht1991
Thanks Received: 1
Vinny Gambini
Vinny Gambini
 
Posts: 6
Joined: August 05th, 2012
 
 
 

Re: Diagram

by eht1991 Tue Aug 14, 2012 8:28 pm

Hello all,

I am brand new to this forum, so please bear with me!

Anyway, just out of curiosity (and for my own reassurance), I wanted to ask if anybody else found this game somewhat tricky to approach?

To me, it seemed as if a lot of crucial inferences were scaffolded onto other inferences, and if you didn't get them all from the start it was impossible to eliminate answer choices. Second, this game combines aspects of numbered ordering, closed assignment, and conditional logic. Is it typical for games on post-2000 exams to be this hybrid?

Thanks!
 
littletiger
Thanks Received: 0
Vinny Gambini
Vinny Gambini
 
Posts: 4
Joined: February 17th, 2015
 
 
 

Re: Diagram

by littletiger Mon Apr 27, 2015 6:57 pm

Anyway, just out of curiosity (and for my own reassurance), I wanted to ask if anybody else found this game somewhat tricky to approach?


I found this game difficult because I had never seen anything like it before (which isn't saying much since I haven't played many games) and consequently did not have an efficient system of notation for these rules. This is why we take practice tests! We'll be much better prepared next time :) .
 
Demi
Thanks Received: 0
Vinny Gambini
Vinny Gambini
 
Posts: 7
Joined: April 01st, 2015
 
 
 

Re: Diagram

by Demi Sun Jan 17, 2016 8:54 pm

would it make sense to use the logic chain for part of our game board?

Thanks!
User avatar
 
ohthatpatrick
Thanks Received: 3808
Atticus Finch
Atticus Finch
 
Posts: 4661
Joined: April 01st, 2011
 
 
 

Re: Diagram

by ohthatpatrick Tue Jan 26, 2016 2:29 pm

Yeah, it could certainly be useful, because there are 4 conditionals and P, F, and H appear in more than one each.

Only the last 3 conditionals have that In/Out feel that goes with the logic chain.

You'd basically get

G -> F and P -> T and H and L .... with all P - T and all H - L

If G is in, everyone is in. So if anyone else is OUT, then G must also be OUT.

You also get
F -> P and T -> H and L .... with all P - T and all H - L

I think the key inference in this game is framing the three possibilities for numerical distributions:
6 ppl in ... each one goes once
5 IN / 1 OUT ... one person goes twice
4 IN / 2 OUT ... two people go twice or one person goes three times

From that big G chain, we know that any time G is in, all 6 ppl are in. So anytime someone is going more than once, we know G is out.

When 5 IN / 1 OUT, then G must be OUT. If anyone else were OUT, then G would ALSO have to be OUT.

When F is in, 5 ppl (at least) are in. So when 4 IN / 2 OUT, then F and G are the two that are OUT.
 
JamieH298
Thanks Received: 0
Vinny Gambini
Vinny Gambini
 
Posts: 1
Joined: July 13th, 2017
 
 
 

Re: Diagram

by JamieH298 Thu Jul 27, 2017 1:31 pm

I'm having a little bit of trouble figuring out a scenario where H can ever be 1st with P 6th in accordance with the last constraint. Does the P preceding T constraint only apply when F has left a message? So, for example, would this arrangement be allowed in order: H-L-T-P-P-P ?